Spherical Harmonics: Proving Y_L^M(0,phi)

Click For Summary
The discussion focuses on proving the equation Y_{L}^{M}(0,φ) = (2L+1)/(4π)^{1/2}δ_{M,0}. The relevant spherical harmonics equation is provided, along with an integral property that highlights orthogonality. The user expresses uncertainty about the integration process needed to combine the equations effectively. Ultimately, the user indicates that they have successfully proven the claim when formatted in LaTeX. The conversation centers on the mathematical proof of spherical harmonics at specific angles.
Elliptic
Messages
33
Reaction score
0

Homework Statement



Prove that

{Y_{L}^{M}\left ( 0,\varphi \right )=\left ( \frac{2L+1}{4\pi } \right )^{1/2}\delta _{M,0}

Homework Equations



Y_{L}^{M}\left ( \theta,\varphi \right )=\left ( \frac{(2L+1)(L-M)!}{4\pi(L+M)! } \right )^{1/2}P_{L}^{M}(cos\theta )e^{im\varphi }

\int_{\varphi =0}^{2\pi }\int_{\theta =0}^{\pi }Y_{L1}^{M1}\left ( \theta ,\varphi \right )Y_{L2}^{M2}\left ( \theta,\varphi \right )sin\theta d\theta d\varphi = \delta _{N1,N2}\delta _{M1,M2}

The Attempt at a Solution



I think i need integrate/combine relevant equations in first equation,but ...?
 

Attachments

  • CodeCogsEqn.gif
    CodeCogsEqn.gif
    5.9 KB · Views: 610
Last edited:
Physics news on Phys.org
written in latex
 
I have proved the claim. Task completed.
 
Question: A clock's minute hand has length 4 and its hour hand has length 3. What is the distance between the tips at the moment when it is increasing most rapidly?(Putnam Exam Question) Answer: Making assumption that both the hands moves at constant angular velocities, the answer is ## \sqrt{7} .## But don't you think this assumption is somewhat doubtful and wrong?

Similar threads

Replies
3
Views
2K
  • · Replies 5 ·
Replies
5
Views
2K
  • · Replies 6 ·
Replies
6
Views
2K
  • · Replies 1 ·
Replies
1
Views
2K
  • · Replies 1 ·
Replies
1
Views
1K
  • · Replies 8 ·
Replies
8
Views
2K
Replies
1
Views
2K
Replies
1
Views
2K
Replies
8
Views
1K